Fourierentwicklung einer einfachen Funktion

Neue Frage »

1.AVM Auf diesen Beitrag antworten »
Fourierentwicklung einer einfachen Funktion
Hallo!

Ich habe hier eine neue Aufgabe, bei der ich keinen gescheiten Anfang (konkreten) finde.

Die Aufgabe lautet:

y = pi² - x²

Im Intervall zwischen -Pi und +Pi

Dazu soll ich eine Fourierentwicklung angeben.

Mein Ansatz:

bisher keiner, aus die allgemeinen Ausdrücke:

a0 = 1/Pi [Integral von -Pi bis +Pi] f(x)dx
a_n = 1/Pi [Integral von -Pi bis +Pi] [f(x)cos (n*x)]dx
b_n = 1/Pi [Integral von -Pi bis +Pi] [f(x)sin (n*x)]dx
Equester Auf diesen Beitrag antworten »

Mal dir mal das Schaubild dazu.
Dann kannst du eine Symmetrie erkennen und die Rechnung vereinfacht sich.

Einfach den entsprechenden allgemeinen Ansatz nehmen und losrechnen Augenzwinkern .
1.AVM Auf diesen Beitrag antworten »

ich habe leider keinen "allgemeinen Ansatz" ... hast du mal nen Link, wo so ein allg. Ansatz bei einer einfachen Aufgabe gezeigt wird?
Equester Auf diesen Beitrag antworten »

Mit "allgemeiner Ansatz" meinte ich eigentlich genau das, was du schon bei deinem
Ansatz gebracht hattest? Du musst nur f(x) dort ersetzen und ausrechnen.

Davor solltest du aber erst einmal zeichnen. Für a_n oder b_n kannst du nämlich
dann direkt eine Aussage machen!
1.AVM Auf diesen Beitrag antworten »

a_n und b_n sind null?
1.AVM Auf diesen Beitrag antworten »

und dann müsste ich die funktion einfach nur integrieren??
 
 
Equester Auf diesen Beitrag antworten »

Naja, wenn a_n und b_n beide 0 wären, wärst du fertig^^.


Kennst du nicht die Eigenschaft, dass bei Punktsymmetrie a_n=0 ist und bei
Achsensymmetrie b_n=0? Damit sparst du dir einiges an Rechnung.

Klären wir erst mal diesen Schritt: Welche Symmetrie liegt vor?
1.AVM Auf diesen Beitrag antworten »

Achssymmetrie zur Y-Achse, also b_n = 0, richtig?


Und dann nur die Funktion im Intervall integrieren?
Equester Auf diesen Beitrag antworten »

Genau...b_n=0

Bleibt also die Bestimmung von:
a0 = 1/Pi [Integral von -Pi bis +Pi] f(x)dx
a_n = 1/Pi [Integral von -Pi bis +Pi] [f(x)cos (n*x)]dx

Wobei du ja über die Symmetrie Bescheid weißt und auch
a0 = 2/Pi [Integral von 0 bis +Pi] f(x)dx
a_n = 2/Pi [Integral von 0 bis +Pi] [f(x)cos (n*x)]dx

bestimmen kannst smile .
1.AVM Auf diesen Beitrag antworten »

Hi,

bei der Aufgabe soll man noch nach gleichmäßiger und absoulter Konvergenz untersuchen, wie mache ich das denn?
Equester Auf diesen Beitrag antworten »

Sry, da kann ich dir nicht helfen.
Die Begriffe (zumindest der erste) sagen mir nichts.

Die Fourierreihenentwicklung aber hat funktioniert? Augenzwinkern
1.AVM Auf diesen Beitrag antworten »

also a_0 habe ich ausrechnen können mit den Grenzen von -P bis Pi, da kommt 4/3 Pi als Ergebnis raus.

Gerade bin ich bei den a_n, da muss man mehrere Male partiell integrieren, was ganz schön Konzentration kostet Big Laugh
Equester Auf diesen Beitrag antworten »

Ich hoffe du meinst Augenzwinkern .

Ja, da brauchts 1-2 mal die partielle Integration, bei der Bestimmung von .
1.AVM Auf diesen Beitrag antworten »

ja *gg*

und wenn ich a_n berechne, bekomme ich einen ausdruck mit n, was bringt mir das, bzw. wie mache ich danach weiter? setze ich das in diese gleichung ein mit a0/2 ... usw.?
Equester Auf diesen Beitrag antworten »

Du hast doch die Form

Du hast nun die Koeffizienten zu bestimmen, wobei du schon richtigerweise als 0 bezeichnet hast.
hattest du gerade ausgerechnet. Für den ersten Summanden musst du dein
Ergebnis also nur noch durch 2 divideren Augenzwinkern .

Bleibt noch der Koeffizient . Da kommt nun deine Integration ins Spiel.
Die Lösung des Koeffizienten musst du später oben einfach nur noch austauschen Augenzwinkern .
1.AVM Auf diesen Beitrag antworten »

also ist a_0 = 2/3 Pi²

auf den Ausdruck mit a_n komme ich nicht, das wird eine ellenlange Integration, Wolfram Alpha meint dazu:

http://www.wolframalpha.com/input/?i=Int...%28cos%28n*x%29

Wie schreibe ich das dann auf?

Schreibe ich das wieder als Ausdruck mit f(x) = (4/3 * Pi²) / 2 + Summe aus (Ergebnis von a_n) + 0 ?
Equester Auf diesen Beitrag antworten »

a_0 = 2/3 Pi²

Das stimmt nicht, das ist a_0/2.


Ja, das Ergebnis von wolfram ist das gesucht. Wenn du die Grenzen einsetzt vereinfacht
sich das ganze aber noch. Für n aus N kannst du dann Aussagen treffen, die den
Bruch zusammenschmelzen lassen Augenzwinkern .


-> f(x) = (4/3 * Pi²) / 2 + Summe aus (Ergebnis von a_n) + 0 ?

Da fehlt noch der cos(nx), sonst aber passts. a_n war ja nur der Koeffizient Augenzwinkern .
Neue Frage »
Antworten »



Verwandte Themen

Die Beliebtesten »
Die Größten »
Die Neuesten »